LSAT and Law School Admissions Forum

Get expert LSAT preparation and law school admissions advice from PowerScore Test Preparation.

 Administrator
PowerScore Staff
  • PowerScore Staff
  • Posts: 8916
  • Joined: Feb 02, 2011
|
#73291
Complete Question Explanation

Flaw in the Reasoning. The correct answer choice is (D).

The presence of conditional reasoning might lead you to suspect that there is a Mistaken Reversal or Mistaken Negation present, but there is no such conditional flaw. Instead, we see a somewhat rare Time Shift error, in which the author uses evidence about what has happened so far (economists have always failed at accurately predicting global recessions) as if it proves that the same thing will always be true (they never will be able to accurately predict those recessions).

Accepting the conditional premise, we get that prevention requires prediction:

Prevent Recession :arrow: Predict Recession

The conclusion that we will never be able to prevent them would be valid if we knew that we could never accurately predict them. That would be a valid contrapositive. But because the evidence is based only on the past and present, and fails to account for possible changes in the future, the conclusion is flawed. The conditional relationship is interpreted correctly, but the evidence of the past is treated as if it is also evidence of the future, the essence of a Time Shift.

Answer choice (A): A description of circular reasoning, which did not occur in the stimulus. Never pick this answer if the author actually provided any evidence to support their claim, even if that evidence is bad.

Answer choice (B): It is irrelevant what economists claim - they might agree with the author that prevention is not possible, or they might disagree and claim that they can do it. The problem is the author's use of evidence of past performance to predict future results.

Answer choice (C): This answer uses "required" and "assures," synonyms for Necessary and Sufficient, and describes a Mistaken Reversal. As discussed above, while there is a conditional premise, the flaw is not based on conditional reasoning, and this answer is a very attractive loser.

Answer choice (D): This is the correct answer choice. This correct answer points out the possibility, overlooked by the author, that things could change in the future.

Answer choice (E): A misstatement of the facts in the stimulus, this answer describes something that did not happen in the argument. The author did not at any point claim, or even imply, that because a recession cannot be predicted, a recession cannot actually happen. Quite the opposite, really - the author seems convinced that recessions WILL occur but will not be accurately predicted.
 Osa
  • Posts: 9
  • Joined: Jun 02, 2015
|
#18856
I think this a circular reasoning. Would Option A have been correct if the statement in Option A is reversed? i.e "Presupposes in a conclusion the premise that it purports to establish"

Also, can you explain why D is right. I can only choose D via process of elimination. It just does make sense to me.
 Lucas Moreau
PowerScore Staff
  • PowerScore Staff
  • Posts: 216
  • Joined: Dec 13, 2012
|
#18864
Hello, Osa,

If you were to change answer choice A like that, it wouldn't make very much sense, I'm afraid. :ras: A is talking about circular reasoning, yes, but that's not the flaw in the stimulus. Premises are not established the way conclusions are, premises are accepted to be true.

Meanwhile, answer choice D is correct because the conclusion is saying, essentially, "We can't predict recessions now, so we'll never be able to predict them, so we'll never be able to prevent them." It's that middle part of the sentence that D shoots down - how do you know we won't get significantly better at predicting recessions? ;)

Hope that helps,
Lucas Moreau
 mpoulson
  • Posts: 148
  • Joined: Mar 25, 2016
|
#25996
Hello,

I chose answer E for this question and still fail to recognize why it is wrong. I thought the stimulus was saying that a GR would not be prevented because they aren't predictable. Thus, I thought it made sense to say that this was an illegitimate inference because although it may be unpredictable there could still be a way to prevent it. Can you explain why E is wrong and why D is correct? Thank you.

V/r,

Micah
 Clay Cooper
PowerScore Staff
  • PowerScore Staff
  • Posts: 241
  • Joined: Jul 03, 2015
|
#26000
Hi Micah,

Thanks for your question. You are almost correct about what the stimulus is claiming - that recessions cannot be prevented. It seems like where you go wrong is in understanding what basis the argument offers for this claim. The argument says that scientists consistently fail to predict recessions, even with the best techniques currently available.

But what if those techniques improve? Isn't it possible that we might then be able to predict, and thus open the door to preventing, recessions? This possibility is what makes answer choice D correct.

Answer choice E is incorrect because it does not actually describe what the stimulus is doing; the stimulus does not infer that something will not occur on the basis that that something is unpredictable. Instead, it argues that we cannot prevent what is not predictable.

Hope that helps!
 freddythepup
  • Posts: 34
  • Joined: Jul 12, 2018
|
#49443
Hi, for this question, I understand why D works, but I was also stuck on C. To me the argument had another flaw in that the premise had Preventable -- > Predictable.

But the conclusion says economists fail to ACCURATELY predict recessions. I thought this was not what was originally given in the premise, which mentions "predictable", but accurately or not accurately predict.

Therefore, I thought C could also work because it is saying the original rule: Preventable --> Predictable , and that being predictable ensures preventable.

can you explain why C does not work or if I'm thinking of this correctly? Thanks!
 Adam Tyson
PowerScore Staff
  • PowerScore Staff
  • Posts: 5153
  • Joined: Apr 14, 2011
|
#49705
Thanks for the question, Freddy! If we apply the details of the stimulus to the abstract description in answer C, here's what we get:

Treats the predictability of a recession, which is necessary in order to prevent it, as being sufficient to prevent it.

You might guess from my choice of words that answer C is describing a conditional issue, and you would be correct. Answer C is saying that the author starts off with a claim that prevention requires prediction, and then concludes that prediction requires prevention. A mistaken reversal!

That's of course not what happened in the stimulus, for two reasons. First, what he did looked more like a contrapositive than a mistaken reversal - if we are to prevent it, we must predict it, but we cannot predict it so we cannot prevent it. But that's not quite what he said either, is it? Instead, he said something more like "if we are to prevent it, we must predict it, but so far and with the best techniques we have we have failed to predict, so we will never be able to prevent. That is neither a valid contrapositive nor a mistaken reversal, but a time shift flaw! We have not succeeded, so we will never succeed.

Answer C is hard to see as a conditional answer, perhaps, but the words "required" and "assures" are synonyms for necessary and sufficient, and that should clue you in to look into whether the flaw is in fact a conditional one. It's not, so C is out.
 medialaw111516
  • Posts: 80
  • Joined: Dec 11, 2018
|
#72055
Why is D better than C? Is D correct because of the phrase "can never" which ignores that just because they can't currently solve it with the techniques at their disposal doesn't mean they'll never be able to
 Jeremy Press
PowerScore Staff
  • PowerScore Staff
  • Posts: 1000
  • Joined: Jun 12, 2017
|
#72095
Hi medialaw,

For the reasons Adam stated in his post above, answer choice C is an inaccurate description of the reasoning and could never be an answer to the question (even if D were not an answer choice). In other words, the author didn't treat predictability as sufficient to prevent recession. Rather, the author treated (current) unpredictability as sufficient to prove (current+future) inability to prevent recession.

You're exactly right about why answer choice D is correct. Just because economists' currently best techniques don't allow them to accurately predict recessions doesn't mean that they wouldn't be able to if they had better techniques.

I hope this helps!

Jeremy
 medialaw111516
  • Posts: 80
  • Joined: Dec 11, 2018
|
#72223
Ugh ok I see it now. I always forget about time shift errors. Thanks Jeremey!

Get the most out of your LSAT Prep Plus subscription.

Analyze and track your performance with our Testing and Analytics Package.